Somma di una serie telescopica – esercizio

Vogliamo calcolare  la somma   S_n:0->oo   1/(n^2 + 7n + 12)

Sappiamo che é convergente perché é confrontabile con S_n:1->oo  1/n^2

che converge in quanto é una p-serie con p > 1.

Operiamo una decomposizione in fratti semplici sul termine generale

1/((n+3)(n+4)) = A/(n+3) + B/(n+4)

An + 4A + Bn + 3B = 1   per ogni n si scinde in

A + B = 0

4A + 3B = 1

per cui segue che

B = -A e  4A – 3A = 1 =>   A = 1 e B = -1

e la nostra serie diventa

S_n:0->oo   (1/(n+3) – 1/(n+4))

fino all’indice N ne risulta la somma parziale SN data da

(1/3 – 1/4) + (1/4 – 1/5 ) + (1/5 – 1/6) + ….+

+ (1/(N+2) – 1/(N+3)) + (1/(N+3) – 1/(N+4))

( che ha la tipica forma a cannocchiale e perciò si chiama telescopica )

ed elidendo gli addendi con segno opposto si deduce infine

SN = 1/3 – 1/(N + 4)

e così S = lim_N->oo   SN = 1/3 – lim_N->oo 1/(N+4) = 1/3 – 0 = 1/3.

 

Nota.

Ho voluto impostare la discussione con un esempio giusto perché si

capisse qualcosa. Per chi ha voglia di generalizzare si può dimostrare,

utilizzando lo stesso identico ragionamento ma con un pò di astrazione

in più, che

S_n:0->oo   1/((n+a)(n+b)) = 1/d S_k:0->d-1    1/(a+k)

essendo d = b – a.

 

Lascia un commento

Il tuo indirizzo email non sarà pubblicato. I campi obbligatori sono contrassegnati *